Đến nội dung

Hoang Tung 126 nội dung

Có 1000 mục bởi Hoang Tung 126 (Tìm giới hạn từ 30-04-2020)



Sắp theo                Sắp xếp  

#473944 Topic về Bất đẳng thức, cực trị THCS

Đã gửi bởi Hoang Tung 126 on 30-12-2013 - 16:38 trong Bất đẳng thức và cực trị

Cho a,b,c>0 .CMR:

 

$\sum \frac{(b+c-a)^{2}}{(b+c)^{2}+a^{2}}\geq \frac{3}{5}$

Bài này có nhiều cách làm Mình xin nêu ra cách ngắn gọn nhất .

Chuẩn hóa :$a+b+c=3$

BĐT $< = > \sum \frac{a(b+c)}{a^2+(b+c)^2}\leq \frac{6}{5}< = > \sum \frac{a(3-a)}{a^2+(3-a)^2}\leq \frac{6}{5}< = > \sum \frac{1}{2a^2-6a+9}\leq \frac{3}{5}$

Mặt khác ta lại có :$\frac{1}{2a^2-6a+9}\leq \frac{2a+3}{25}< = > a^3+a^3+1\geq 3a^2$(Luôn đúng theo AM-GM 3 số)

$= > \sum \frac{1}{2a^2-6a+9}\leq \frac{2\sum a+9}{25}=\frac{2.3+9}{25}=\frac{3}{5}$(đpcm)

 Dấu = xảy ra khi $a=b=c=1$




#475064 Topic về Bất đẳng thức, cực trị THCS

Đã gửi bởi Hoang Tung 126 on 03-01-2014 - 20:37 trong Bất đẳng thức và cực trị

Vậy giả sử a+b+c=1 cũng được ạ?

Tùy thôi miễn là tổng của 3 số không âm là được




#474808 Topic về Bất đẳng thức, cực trị THCS

Đã gửi bởi Hoang Tung 126 on 02-01-2014 - 20:12 trong Bất đẳng thức và cực trị

Chỗ này chuẩn hoá nghĩa là sao???

Có nghĩa là đề bài cho 3 ẩn thì mình có thể giả sử được tổng của 3 ẩn ấy 




#482721 $\boxed{\text{Chuyên Đề}}$ Bất đẳng thức - Cực trị

Đã gửi bởi Hoang Tung 126 on 12-02-2014 - 16:37 trong Bất đẳng thức và cực trị

Bài 42:Theo AM-GM có:$\sum \frac{1}{a^2+bc}\leq \sum \frac{1}{2a\sqrt{bc}}=\frac{\sum \sqrt{bc}}{2abc}\leq \frac{\sum a}{2abc}$

Dấu = xảy ra tại a=b=c=1




#482720 $\boxed{\text{Chuyên Đề}}$ Bất đẳng thức - Cực trị

Đã gửi bởi Hoang Tung 126 on 12-02-2014 - 16:36 trong Bất đẳng thức và cực trị

41) Cho $x;y;z>0$. Cmr: $\sum\frac{2\sqrt{x}}{x^3+y^2}\leq \sum \frac{1}{x^2}$

 

42) Cho $a;b;c>0$. Cmr: $\sum \frac{1}{a^2+bc}\leq \frac{a+b+c}{2abc}$

 

43) Cho $\left\{\begin{matrix}x;y;z>0 & & \\ x+y+z=1 & & \end{matrix}\right.$. Cmr: $\sum \sqrt{x^2+xy+y^2}\geq \sqrt{3}$

 

44) Cho $\left\{\begin{matrix}x;y;z>0 & & \\ x+y+z=1 & & \end{matrix}\right.$. Cmr: $\sum \sqrt{2x^2+xy+y^2}\geq \sqrt{5}$

 

45) Cho $a;b>0$. Tìm Max $Q=a^3+b^3$ biết $a+b=a^2-ab+b^2$ 

 

46) Cho $a;b;c>0$ thỏa $a^2+b^2+c^2=1$. Cmr: $\sum \frac{1}{a^2+b^2}\leq \frac{\sum a^3}{2abc}+3$

Bài 41:Theo AM-GM có:$\sum \frac{2\sqrt{x}}{x^3+y^2}\leq \sum \frac{2\sqrt{x}}{2\sqrt{x^3y^2}}=\sum \frac{1}{xy}\leq \sum \frac{1}{x^2}$

Đẳng thức xảy ra tại $x=y=z=1$




#482236 $\boxed{\text{Chuyên Đề}}$ Bất đẳng thức - Cực trị

Đã gửi bởi Hoang Tung 126 on 09-02-2014 - 16:00 trong Bất đẳng thức và cực trị

Bài 7: BĐT $< = > (\sum a)(\sum \frac{1}{a})-3\geq 3\sqrt[3]{\frac{3(a+b+c)(a+b)(b+c)(c+a)}{(ab+bc+ac)^2}}< = > \sum \frac{a+b}{c}\geq 3\sqrt[3]{\frac{3(a+b+c)(a+b)(b+c)(c+a)}{(ab+bc+ac)^2}}$

Mặt khác theo AM-GM 3 số có:$\sum \frac{a+b}{c}\geq 3\sqrt[3]{\frac{(a+b)(b+c)(c+a)}{abc}}$

 Do đó ta cần CM:$3\sqrt[3]{\frac{(a+b)(b+c)(c+a)}{abc}}\geq 3\sqrt[3]{\frac{3(a+b+c)(a+b)(b+c)(c+a)}{(ab+bc+ac)^2}}< = > (ab+bc+ac)^2\geq 3abc(a+b+c)< = > (ab-ac)^2+(bc-ba)^2+(ca-cb)^2\geq 0$

(Luôn đúng)

 Do đó ta có ĐPCM. Dấu = xảy ra tại $a=b=c$




#482744 $\boxed{\text{Chuyên Đề}}$ Bất đẳng thức - Cực trị

Đã gửi bởi Hoang Tung 126 on 12-02-2014 - 17:40 trong Bất đẳng thức và cực trị

Daicagiangho1998 nhớ trích dẫn đề nha.

 

47) Cho $a;b;c>0$ thỏa $abc=2$. Cmr: $\sum a^3\geq \sum a\sqrt{b+c}$

 

48) Cho $a;b>0$ thỏa $a^2+b^2=1$. Tìm Min $A=\frac{1}{a}+\frac{1}{b}-(\sqrt{\frac{a}{b}-\sqrt{\frac{b}{a}}})^2$

49) Cho 2 dãy ${a_n};{b_n}>0$ thoả: $a_{n+1}=a_n+\frac{1}{a_n}$ ; $b_{n+1}=b_n+\frac{1}{b_n}$.

Cmr: $C_{25}=a_{25}+b_{25}>10\sqrt{2}$

 

50) Cho $a;b>0$. Cmr:

$\sqrt{ab}+\frac{(a-b)^2(3a+b)(a+3b)}{8(a+b)(a^2+6ab+b^2)}\leq \frac{a+b}{2}$

 

51) Cho $a;b;c>0$ thoar: $a^2+b^2+c^2=4\sqrt{abc}$

Cmr: $a+b+c>2\sqrt{abc}$

Bài 50:BDT $< = > \frac{a+b}{2}\geq \sqrt{ab}+\frac{(a-b)^2(3a+b)(a+3b)}{8(a+b)(a^2+6ab+b^2)}< = > (\sqrt{a}-\sqrt{b})^6\geq 0$(Luôn đúng)




#482658 $\boxed{\text{Chuyên Đề}}$ Bất đẳng thức - Cực trị

Đã gửi bởi Hoang Tung 126 on 12-02-2014 - 09:19 trong Bất đẳng thức và cực trị



 

31) Cho $\left\{\begin{matrix}a_1;a_2;...;a_n>0 & & \\ a_1+a_2+...+a_n=1 & & \end{matrix}\right.$. Cmr: $(\frac{1}{a_1}-1)(\frac{1}{a_2}-1)...(\frac{1}{a_n}-1)\geq (n-1)^n$

 

32) Cho $x+y+z=0$. Tìm Min $P=\sum \sqrt{3+4^x}$

 

33) Cho $a;b;c>0$ thỏa $a^2+b^2+c^2=3$. Tìm Min $A=\sum \frac{a^3}{\sqrt{b^2+3}}$

 

34) Cho $a;b;c>0$ thỏa: $a+b+c=\frac{3}{4}$. Tìm Min $P=\sum \frac{1}{\sqrt[3]{a+3b}}$

 

35) Cho $x;y;z>0$ thỏa: $xyz=1$. Tìm Min $A=\sum \frac{x^2(y+z)}{y\sqrt{y}+2z\sqrt{z}}$

 

 Còn mỗi bài này mình giải luôn

Bài 35: $\sqrt{x}=a,\sqrt{y}=b,\sqrt{z}=c= > abc=\sqrt{xyz}=1$.Ta sử dụng bdt Bunhiacopxki và bdt AM-GM như sau:

$= > A=\sum \frac{a^4(b^2+c^2)}{b^3+2c^3}\geq \sum \frac{a^4.2bc}{b^3+2c^3}=\sum \frac{2a^3.abc}{b^3+2c^3}=\sum \frac{2a^3}{b^3+2c^3}=2\sum \frac{a^3}{b^3+2c^3}=2\sum \frac{a^6}{a^3b^3+2a^3c^3}\geq 2.\frac{(\sum a^3)^2}{3\sum a^3b^3}=\frac{2}{3}.\frac{(\sum a^3)^2}{\sum a^3b^3}\geq \frac{2}{3}.\frac{3\sum a^3b^3}{\sum a^3b^3}=2= > A\geq 2$

Do đó A Min = 2 khi $a=b=c=1< = > x=y=z=1$




#482911 $\boxed{\text{Chuyên Đề}}$ Bất đẳng thức - Cực trị

Đã gửi bởi Hoang Tung 126 on 13-02-2014 - 18:17 trong Bất đẳng thức và cực trị

52) Cho $\left\{\begin{matrix}a;b;c>0 & & \\ a+b+c=4 & & \end{matrix}\right.$. Cmr: $4\leq \sum \sqrt{a+b}\leq 2\sqrt{6}$

 

53) Cho $a;b;c$ là độ dài 3 cạnh một tam giác, Cmr: $\sum \sqrt{a+b-c}\leq \sum \sqrt{a}$

 

54) Cho $a;b;c$ là độ dài 3 cạnh một tam giác có chu vi bằng $2$. Cmr: $\sum a^2<2(1-abc)$

 

55) Xác định dạng tam giác $ABC$ có chu vi bằng $1$ biết $A=\sum \frac{a}{1-2a}$ đạt $min$.

 

56) Cho $a;b;c>0$ thỏa $a^2+b^2+c^2=1$. Cmr: $\sum \frac{a^2}{1+b-a}\geq 1$

 

57) Cho $a;b;c$ là độ dài 3 cạnh một tam giác. Cmr: $\sum \sqrt[3]{a+b-c}\leq \sum \sqrt[3]{a}$

Bài 52:Theo Bunhiacopxki có:$(\sum \sqrt{a+b})^2\leq 3(2\sum a)=6\sum a=6= > \sum \sqrt{a+b}\leq \sqrt{6}$

Đẳng thức xảy ra tại $a=b=c=\frac{1}{3}$

Ta có:$a+b+c=4= > a+b< 4= > (a+b)(a+b-4)< 0= > \frac{(a+b)^2}{4}< a+b= > \frac{a+b}{2}< \sqrt{a+b}$

Tương tự $\frac{b+c}{2}< \sqrt{b+c},\frac{c+a}{2}< \sqrt{c+a}$

Cộng theo vế $= > \sum \sqrt{a+b}> \sum \frac{a+b}{2}=\sum a=4$




#483034 $\boxed{\text{Chuyên Đề}}$ Bất đẳng thức - Cực trị

Đã gửi bởi Hoang Tung 126 on 14-02-2014 - 08:28 trong Bất đẳng thức và cực trị

58) Cho $x\in (0;1)$. Tìm Min $A=\frac{4x^2+1}{x^2(1-x)}$

 

59) Cho $x\in (0;3)$. Tìm Max $B=(5x^2-14x-3)(x-3)$

 

60) Cho $a;b;c$ là độ dài 3 cạnh một tam giác. Cmr: $\sum \frac{a}{\sqrt{b+c-a}}\geq \sum \sqrt{a}$

 

61) Cho $a;b;c$ là độ dài 3 cạnh một tam giác. Cmr: $|\sum \frac{a-b}{a+b}|<\frac{1}{8}$

 

62) Cho $x\in [0;1]$. Cmr: $\sqrt[4]{1-x^2}+\sqrt[4]{1-x}+\sqrt[4]{1+x}\leq 3$

 

63) Cho $a;b;c>0$. Cmr: $\frac{1}{a^3}+\frac{a^3}{b^3}+b^3\geq \frac{1}{a}+\frac{a}{b}+b$

 

64)

a) Cho $x\in [-1;1]$. Cmr: $|4x^3-3x|\leq 1$

b) Cho $\left\{\begin{matrix}a_1;a_2;...;a_n\in [-1;1] & & \\ a_1^3+a_2^3+...+a_n^3=0 & & \end{matrix}\right.$. Cmr: $a_1+a_2+...+a_n\leq \frac{n}{3}$

 

65) Cho $x;y>0$ thỏa $x^2+y^2=1$. Cmr: $xy+Max(x;y)\leq \frac{3\sqrt{3}}{4}$ (Giải thích: Nếu $x>y$ thì $Max(x;y)=x$ và tương tự)

 

 

P/s: Anh Daicagiangho1998 học KHTN nên cứ từ từ mà làm thôi, chứ có đề phát đã ăn hết sạch luôn vậy :D

Bài 64:Ta có:$\left | 4x^3-3x \right |\leq 1< = > -1\leq 4x^3-3x\leq 1< = > 4x^3-3x+1\geq 0,1+3x-4x^3\geq 0< = > (2x-1)^2(x+1)\geq 0,(1-x)(4x^2-4x+1)\geq 0$(Luôn đúng theo giả thiết)

 

 

 



#483033 $\boxed{\text{Chuyên Đề}}$ Bất đẳng thức - Cực trị

Đã gửi bởi Hoang Tung 126 on 14-02-2014 - 08:18 trong Bất đẳng thức và cực trị

58) Cho $x\in (0;1)$. Tìm Min $A=\frac{4x^2+1}{x^2(1-x)}$

 

59) Cho $x\in (0;3)$. Tìm Max $B=(5x^2-14x-3)(x-3)$

 

60) Cho $a;b;c$ là độ dài 3 cạnh một tam giác. Cmr: $\sum \frac{a}{\sqrt{b+c-a}}\geq \sum \sqrt{a}$

 

61) Cho $a;b;c$ là độ dài 3 cạnh một tam giác. Cmr: $|\sum \frac{a-b}{a+b}|<\frac{1}{8}$

 

62) Cho $x\in [0;1]$. Cmr: $\sqrt[4]{1-x^2}+\sqrt[4]{1-x}+\sqrt[4]{1+x}\leq 3$

 

63) Cho $a;b;c>0$. Cmr: $\frac{1}{a^3}+\frac{a^3}{b^3}+b^3\geq \frac{1}{a}+\frac{a}{b}+b$

 

64)

a) Cho $x\in [-1;1]$. Cmr: $|4x^3-3x|\leq 1$

b) Cho $\left\{\begin{matrix}a_1;a_2;...;a_n\in [-1;1] & & \\ a_1^3+a_2^3+...+a_n^3=0 & & \end{matrix}\right.$. Cmr: $a_1+a_2+...+a_n\leq \frac{n}{3}$

 

65) Cho $x;y>0$ thỏa $x^2+y^2=1$. Cmr: $xy+Max(x;y)\leq \frac{3\sqrt{3}}{4}$ (Giải thích: Nếu $x>y$ thì $Max(x;y)=x$ và tương tự)

 

 

P/s: Anh Daicagiangho1998 học KHTN nên cứ từ từ mà làm thôi, chứ có đề phát đã ăn hết sạch luôn vậy :D

Bài 62:Theo AM-GM 4 số có:$\sqrt[4]{1-x^2}=\sqrt[4]{(1-x)(1+x).1.1}\leq \frac{1-x+1+x+1+1}{4}=1$

                                             $\sqrt[4]{(1-x).1.1.1}\leq \frac{1-x+1+1+1}{4}=\frac{4-x}{4}$

                                             $\sqrt[4]{(1+x).1.1.1}\leq \frac{1+x+1+1+1}{4}=\frac{4+x}{4}$

Cộng theo vế các bdt $= > \sqrt[4]{1-x^2}+\sqrt[4]{1+x}+\sqrt[4]{1-x}\leq \frac{4-x+4+x}{4}+1=3$




#482912 $\boxed{\text{Chuyên Đề}}$ Bất đẳng thức - Cực trị

Đã gửi bởi Hoang Tung 126 on 13-02-2014 - 18:20 trong Bất đẳng thức và cực trị

52) Cho $\left\{\begin{matrix}a;b;c>0 & & \\ a+b+c=4 & & \end{matrix}\right.$. Cmr: $4\leq \sum \sqrt{a+b}\leq 2\sqrt{6}$

 

53) Cho $a;b;c$ là độ dài 3 cạnh một tam giác, Cmr: $\sum \sqrt{a+b-c}\leq \sum \sqrt{a}$

 

54) Cho $a;b;c$ là độ dài 3 cạnh một tam giác có chu vi bằng $2$. Cmr: $\sum a^2<2(1-abc)$

 

55) Xác định dạng tam giác $ABC$ có chu vi bằng $1$ biết $A=\sum \frac{a}{1-2a}$ đạt $min$.

 

56) Cho $a;b;c>0$ thỏa $a^2+b^2+c^2=1$. Cmr: $\sum \frac{a^2}{1+b-a}\geq 1$

 

57) Cho $a;b;c$ là độ dài 3 cạnh một tam giác. Cmr: $\sum \sqrt[3]{a+b-c}\leq \sum \sqrt[3]{a}$

Bài 53:Theo Bunhiacopxki có:$\sqrt{a+b-c}+\sqrt{b+c-a}\leq \sqrt{2(a+b-c+b+c-a)}=\sqrt{4b}=2\sqrt{b}$

                                             $\sqrt{a+b-c}+\sqrt{a+c-b}\leq 2\sqrt{a}$

                                             $\sqrt{b+c-a}+\sqrt{a+c-b}\leq 2\sqrt{b}$

Cộng theo vế các pt $= > 2\sum \sqrt{a+c-b}\leq 2\sum \sqrt{a}= > \sum \sqrt{a+c-b}\leq \sum \sqrt{a}$(ĐPCM)




#482226 $\boxed{\text{Chuyên Đề}}$ Bất đẳng thức - Cực trị

Đã gửi bởi Hoang Tung 126 on 09-02-2014 - 15:33 trong Bất đẳng thức và cực trị

Bài 14:Theo AM-GM có:$P=\frac{a}{\sqrt{a(b+2c)}}+\frac{b}{\sqrt{b(c+2a)}}+\frac{c}{\sqrt{c(a+b+c)}}+\frac{c}{\sqrt{c(a+b+c)}}\geq \frac{a}{\frac{a+b+2c}{2}}+\frac{b}{\frac{b+c+2a}{2}}+\frac{c}{\frac{a+b+2c}{2}}+\frac{c}{\frac{a+b+2c}{2}}=2(\frac{a}{a+b+2c}+\frac{b}{b+c+2a}+\frac{c}{a+b+2c}+\frac{c}{a+b+2c})=2(\frac{a^2}{ab+a^2+2ac}+\frac{b^2}{b^2+bc+2ab}+\frac{c^2}{ac+bc+2c^2}+\frac{c^2}{ac+bc+2c^2})\geq 2.\frac{(a+b+2c)^2}{a^2+b^2+4c^2+3ab+3ac+3bc}=2.\frac{a^2+b^2+4c^2+2ab+4ac+4bc}{a^2+b^2+4c^2+3ab+3ac+3bc}\geq 2< = > ac+bc\geq ab$




#482747 $\boxed{\text{Chuyên Đề}}$ Bất đẳng thức - Cực trị

Đã gửi bởi Hoang Tung 126 on 12-02-2014 - 17:46 trong Bất đẳng thức và cực trị

Daicagiangho1998 nhớ trích dẫn đề nha.

 

47) Cho $a;b;c>0$ thỏa $abc=2$. Cmr: $\sum a^3\geq \sum a\sqrt{b+c}$

 

48) Cho $a;b>0$ thỏa $a^2+b^2=1$. Tìm Min $A=\frac{1}{a}+\frac{1}{b}-(\sqrt{\frac{a}{b}-\sqrt{\frac{b}{a}}})^2$

49) Cho 2 dãy ${a_n};{b_n}>0$ thoả: $a_{n+1}=a_n+\frac{1}{a_n}$ ; $b_{n+1}=b_n+\frac{1}{b_n}$.

Cmr: $C_{25}=a_{25}+b_{25}>10\sqrt{2}$

 

50) Cho $a;b>0$. Cmr:

$\sqrt{ab}+\frac{(a-b)^2(3a+b)(a+3b)}{8(a+b)(a^2+6ab+b^2)}\leq \frac{a+b}{2}$

 

51) Cho $a;b;c>0$ thoar: $a^2+b^2+c^2=4\sqrt{abc}$

Cmr: $a+b+c>2\sqrt{abc}$

Bài 51:Theo AM-GM có:$4\sqrt{abc}=a^2+b^2+c^2\geq 3\sqrt[3]{a^2b^2c^2}= > 4^6.(abc)^3\geq 3^6.(abc)^4< = > abc\leq \frac{4^6}{3^6}$

Mà $a+b+c\geq 3\sqrt[3]{abc}\geq 2\sqrt{abc}< = > 3^6.(abc)^2\geq 2^6(abc)^3< = > abc\leq (\frac{3}{2})^6< (\frac{4}{3})^6$(Luôn đúng)




#482221 $\boxed{\text{Chuyên Đề}}$ Bất đẳng thức - Cực trị

Đã gửi bởi Hoang Tung 126 on 09-02-2014 - 15:11 trong Bất đẳng thức và cực trị

Bài 10:Theo bđt AM-GM và bđt Schur bậc 3 có :

  $\sum \sqrt{\frac{a}{b+c-a}}\geq 3\sqrt[6]{\frac{abc}{(a+b-c)(b+c-a)(c+a-b)}}\geq 3\sqrt[6]{\frac{abc}{abc}}=3$




#482012 $\boxed{\text{Chuyên Đề}}$ Bất đẳng thức - Cực trị

Đã gửi bởi Hoang Tung 126 on 08-02-2014 - 19:14 trong Bất đẳng thức và cực trị

23) Cho $\left\{\begin{matrix}a;b;c>0 & & \\ a+b+c=1 & & \end{matrix}\right.$. Cmr: $\sum \frac{1}{a+bc}\geq \frac{27}{4}$

 

24) Cho $\left\{\begin{matrix}a;b;c>0 & & \\ a^3c+b^3a+c^3b=abc & & \end{matrix}\right.$. Cmr: $\sum \frac{b}{a^2+ab}\geq \frac{9}{2}$

 

25) Cho $a;b;c>0$. Cmr: $\sum \frac{ab}{a+b+2c}\leq \frac{\sum a}{4}$

 

26) Cho $a;b;c>0$. Cmr: $\sum \frac{ab}{a+3b+2c}\leq \frac{\sum a}{6}$

Bài 26:Ta có:$\sum \frac{ab}{a+3b+2c}=\sum \frac{ab}{(c+b)+(c+a)+2b}\leq \frac{1}{9}(\sum \frac{ab}{c+b}+\sum \frac{ab}{a+c}+\sum \frac{ab}{2b})=\frac{1}{9}(\sum \frac{ab}{c+b}+\sum \frac{ac}{c+b}+\sum \frac{a}{2})=\frac{\sum a}{6}$




#482011 $\boxed{\text{Chuyên Đề}}$ Bất đẳng thức - Cực trị

Đã gửi bởi Hoang Tung 126 on 08-02-2014 - 19:11 trong Bất đẳng thức và cực trị

23) Cho $\left\{\begin{matrix}a;b;c>0 & & \\ a+b+c=1 & & \end{matrix}\right.$. Cmr: $\sum \frac{1}{a+bc}\geq \frac{27}{4}$

 

24) Cho $\left\{\begin{matrix}a;b;c>0 & & \\ a^3c+b^3a+c^3b=abc & & \end{matrix}\right.$. Cmr: $\sum \frac{b}{a^2+ab}\geq \frac{9}{2}$

 

25) Cho $a;b;c>0$. Cmr: $\sum \frac{ab}{a+b+2c}\leq \frac{\sum a}{4}$

 

26) Cho $a;b;c>0$. Cmr: $\sum \frac{ab}{a+3b+2c}\leq \frac{\sum a}{6}$

Bài 24:Ta có:$\sum \frac{ab}{a+b+2c}=\sum \frac{ab}{(a+c)+(a+c)}\leq \frac{1}{4}(\sum \frac{ab}{a+c}+\sum \frac{ab}{a+c})=\frac{1}{4}(\sum \frac{ab}{a+c}+\sum \frac{bc}{a+c})=\frac{1}{4}(\sum a)$




#482009 $\boxed{\text{Chuyên Đề}}$ Bất đẳng thức - Cực trị

Đã gửi bởi Hoang Tung 126 on 08-02-2014 - 19:09 trong Bất đẳng thức và cực trị

23) Cho $\left\{\begin{matrix}a;b;c>0 & & \\ a+b+c=1 & & \end{matrix}\right.$. Cmr: $\sum \frac{1}{a+bc}\geq \frac{27}{4}$

 

24) Cho $\left\{\begin{matrix}a;b;c>0 & & \\ a^3c+b^3a+c^3b=abc & & \end{matrix}\right.$. Cmr: $\sum \frac{b}{a^2+ab}\geq \frac{9}{2}$

 

25) Cho $a;b;c>0$. Cmr: $\sum \frac{ab}{a+b+2c}\leq \frac{\sum a}{4}$

 

26) Cho $a;b;c>0$. Cmr: $\sum \frac{ab}{a+3b+2c}\leq \frac{\sum a}{6}$

Bài 23:Ta có:$\sum \frac{1}{a+bc}\geq \frac{9}{\sum a+\sum bc}\geq \frac{9}{1+\frac{(\sum a)^2}{3}}=\frac{9}{1+\frac{1}{3}}=\frac{27}{4}$




#482007 $\boxed{\text{Chuyên Đề}}$ Bất đẳng thức - Cực trị

Đã gửi bởi Hoang Tung 126 on 08-02-2014 - 19:06 trong Bất đẳng thức và cực trị

 

Bài này làm chưa hoàn chỉnh đâu nhé.

22)

$\sqrt{1+a^3}=\sqrt{(1+a)(a^2-a+1)}\leq \frac{a^2+2}{2}$ (Cauchy)

$\Leftrightarrow \frac{1}{\sqrt{1+a^3}}\geq \frac{2}{a^2+2}$

cmtt...

$\Rightarrow \sum \frac{1}{\sqrt{1+a^3}}\geq 2\sum \frac{1}{a^2+2}$

Vậy ta cần CM: $\sum \frac{1}{a^2+2}\geq \frac{1}{2}$

 

Mọi người chứng minh BĐT phụ này đi.

 

 

 

Đặt $a=\frac{2yz}{x^2},b=\frac{2xz}{y^2},c=\frac{2xy}{z^2}$

$= > \sum \frac{1}{a^2+2}=\sum \frac{1}{(\frac{2yz}{x^2})^2+2}=\sum \frac{x^4}{2x^4+4y^2z^2}\geq \frac{(\sum x^2)^2}{2(\sum x^4+2\sum y^2z^2}=\frac{(\sum x^2)^2}{2(\sum x^2)^2}=\frac{1}{2}$




#482125 $\boxed{\text{Chuyên Đề}}$ Bất đẳng thức - Cực trị

Đã gửi bởi Hoang Tung 126 on 09-02-2014 - 09:32 trong Bất đẳng thức và cực trị

Bài 27:Theo Bunhiacopxki có:$\sum \sqrt{a^2+b^2}\geq \sum \frac{a+b}{\sqrt{2}}=\sqrt{2}.\sum a$




#482126 $\boxed{\text{Chuyên Đề}}$ Bất đẳng thức - Cực trị

Đã gửi bởi Hoang Tung 126 on 09-02-2014 - 09:33 trong Bất đẳng thức và cực trị

Bài 28:Ta có:$\sum \sqrt[3]{a^3+b^3}\geq \sum \frac{a+b}{\sqrt[3]{4}}=\sqrt[3]{2}\sum a$




#482208 $\boxed{\text{Chuyên Đề}}$ Bất đẳng thức - Cực trị

Đã gửi bởi Hoang Tung 126 on 09-02-2014 - 14:25 trong Bất đẳng thức và cực trị

 

6)

 

$\sqrt{\frac{a^2+bc}{a(b+c)}.1}\leq \frac{a^2+ab+bc+ca}{2ab+2ac}$
$\Rightarrow \sqrt{\frac{a(b+c}{a^2+bc}}\geq \frac{2ab+2ac}{(a+b)(a+c)}$
$\Rightarrow \sum \sqrt{\frac{a(b+c}{a^2+bc}}\geq 2$

 

Hình như sai rồi




#482193 $\boxed{\text{Chuyên Đề}}$ Bất đẳng thức - Cực trị

Đã gửi bởi Hoang Tung 126 on 09-02-2014 - 13:52 trong Bất đẳng thức và cực trị

Hì, nhầm rồi Daicagiangho1998, giả thiết là $x+y+z+y=2$ mà, nhừng bài của Daicagiangho1998 là $x+y+z+t=1$ rồi. Fix lại đi, $min=16$

 

 

Không thỏa mãn thì tìm ra A làm gì?

Uhm mình nhầm chỗ đó nhưng cách làm vẫn đúng




#482127 $\boxed{\text{Chuyên Đề}}$ Bất đẳng thức - Cực trị

Đã gửi bởi Hoang Tung 126 on 09-02-2014 - 09:34 trong Bất đẳng thức và cực trị

Bài 29:Áp dụng bđt $x^4+y^4\geq \frac{(x+y)^4}{8}$

$= > \sum \sqrt[4]{a^4+b^4}\geq \sum \sqrt[4]{\frac{(a+b)^4}{8}}=\sum \frac{a+b}{\sqrt[4]{8}}=\sqrt[4]{2}\sum a$




#482223 $\boxed{\text{Chuyên Đề}}$ Bất đẳng thức - Cực trị

Đã gửi bởi Hoang Tung 126 on 09-02-2014 - 15:16 trong Bất đẳng thức và cực trị

7) Cho $a;b;c>0$. Cmr: $\sum a.\sum \frac{1}{a}\geq 3[1+\sqrt[3]{\frac{3(a+b+c)(a+b)(b+c)(c+a)}{(ab+bc+ca)^2}}]$

8) Cho $a;b;c>0$. Cmr: $\sum \sqrt[3]{\frac{a^3}{a^3+(b+c)^3}}\geq 1$

9) Cho $a;b;c;d>0$. Cmr: $\sum \sqrt{\frac{a}{b+c+d}} \geq 2$

10) Cho $a;b;c$ là độ dài 3 cạnh 1 tam giác. Cmr: $\sum \sqrt{\frac{a}{b+c-a}}\geq 3$

11) Cho $a;b;c>0$ thỏa: $a+b+c=3$. Cmr: $2(ab+bc+ca)+\sum \frac{1}{ab}\geq 9$

12) Cho $a;b;c>0$ thỏa: $a+b+c=3abc$. Cmr: $\sum \frac{1}{a^3}\geq 3$

13) Cho $a;b;c>0$ thỏa: $abc=1$. Cmr: $\sum a^3\geq 2.\sum \frac{a}{b+c}$

14) Cho $a;b;c>0$ thỏa: $\sqrt{\frac{a}{b+2c}}+\sqrt{\frac{b}{c+2a}}+2\sqrt{\frac{c}{a+b+c}}\geq 2$

Bài 13:Áp dụng bđt AM-GM $= >2 \sum a^3\geq \sum ab(a+b)=\sum \frac{a+b}{c}$(Do abc=1)(1)

Theo bđt $\frac{1}{x}+\frac{1}{y}\geq \frac{4}{x+y}$

$= > \sum \frac{a+b}{c}=\sum \frac{a}{c}+\sum \frac{a}{b}=\sum a(\frac{1}{b}+\frac{1}{c})\geq \sum \frac{4a}{b+c}= > \sum \frac{a+b}{c}\geq \sum \frac{4a}{b+c}$(2)

Từ (1),(2)$= > 2\sum a^3\geq 4\sum \frac{a}{b+c}= > \sum a^3\geq 2\sum \frac{a}{b+c}$